A thin, 52.0 cm long metal bar with mass 750 g rests on, but is not attached to, two metallic supports in a uniform magnetic field with a magnitude of 0.440 T, as shown in ( Figure 1). A battery and a resistor of resistance 29.0 Ω are connected in series to the supports. For related problem-solving tips and strategies, you may want to view a Video Tutor Solution of Magnetic force on a straight conductor. Figure 1 of 1 Part A What is the largest voltage the battery can have without breaking the circuit at the supports? Express your answer in volts. Submit Request Answer Part B The battery voltage has the maximum value calculated in part (a). If the resistor suddenly gets partially short-circuited, decreasing its resistance to 2.00 Ω, find the initial acceleration of the bar. Express your answer in meters per second squared. a = m/s2

A thin, 52.0 cm long metal bar with mass 750 g rests on, but is not attached to, two metallic supports in a uniform magnetic field with a magnitude of 0.440 T, as shown in ( Figure 1). A battery and a resistor of resistance 29.0 Ω are connected in series to the supports. For related problem-solving tips and strategies, you may want to view a Video Tutor Solution of Magnetic force on a straight conductor. Figure 1 of 1 Part A What is the largest voltage the battery can have without breaking the circuit at the supports? Express your answer in volts. Submit Request Answer Part B The battery voltage has the maximum value calculated in part (a). If the resistor suddenly gets partially short-circuited, decreasing its resistance to 2.00 Ω, find the initial acceleration of the bar. Express your answer in meters per second squared. a = m/s2A thin, 52.0 cm long metal bar with mass 750 g rests on, but is not attached to, two metallic supports in a uniform magnetic field with a magnitude of 0.440 T, as shown in ( Figure 1). A battery and a resistor of resistance 29.0 Ω are connected in series to the supports. For related problem-solving tips and strategies, you may want to view a Video Tutor Solution of Magnetic force on a straight conductor. Figure 1 of 1 Part A What is the largest voltage the battery can have without breaking the circuit at the supports? Express your answer in volts. Submit Request Answer Part B The battery voltage has the maximum value calculated in part (a). If the resistor suddenly gets partially short-circuited, decreasing its resistance to 2.00 Ω, find the initial acceleration of the bar. Express your answer in meters per second squared. a = m/s2

Image text
A thin, 52.0 c m long metal bar with mass 750 g rests on, but is not attached to, two metallic supports in a uniform magnetic field with a magnitude of 0.440 T , as shown in ( Figure 1). A battery and a resistor of resistance 29.0 Ω are connected in series to the supports.
For related problem-solving tips and strategies, you may want to view a Video Tutor Solution of Magnetic force on a straight conductor.
Figure 1 of 1
Part A
What is the largest voltage the battery can have without breaking the circuit at the supports?
Express your answer in volts. Submit Request Answer Part B
The battery voltage has the maximum value calculated in part (a). If the resistor suddenly gets partially short-circuited, decreasing its resistance to 2.00 Ω , find the initial acceleration of the bar.
Express your answer in meters per second squared.

Detailed Answer